Những câu hỏi liên quan
Hậuu
Xem chi tiết
iamRinz
4 tháng 1 2023 lúc 18:48

a, Hàm số \(\left(d_1\right)y=-2x+3\)

Cho \(y=0=>x=\dfrac{3}{2}\) ta được điểm \(\left(\dfrac{3}{2};0\right)\)

Cho \(x=0=>y=3\) ta được điểm \(\left(0;3\right)\)

Vẽ đồ thị hàm số \(\left(d_1\right)\) đi qua hai điểm trên

     hàm số \(\left(d_2\right)y=x-1\)

Cho \(y=0=>x=1\) ta được điểm \(\left(1;0\right)\)

Cho \(x=0=>y=-1\) ta được điểm \(\left(0;-1\right)\)

Vẽ đồ thị hàm số \(\left(d_2\right)\) đi qua hai điểm trên

# Bạn có thể tự vẽ nhé !!

b, Tọa độ giao điểm \(\left(d_1\right);\left(d_2\right)\) là nghiệm của pt

\(-2x+3=x-1\\ =>-3x=-4\\ =>x=\dfrac{4}{3}\)

Thay \(x=\dfrac{4}{3}\) vào \(\left(d_2\right)\)

\(\Rightarrow y=\dfrac{4}{3}-1=\dfrac{1}{3}\)

Vậy tọa độ giao điểm là : \(\left(\dfrac{4}{3};\dfrac{1}{3}\right)\)

c, Giả sử \(\left(d_3\right)y=ax+b\)

\(\left(d_3\right)\) đi qua \(A\left(-2;1\right)\) và song song với đường thẳng \(\left(d_1\right)y=-2x+3\)

\(\Rightarrow\left\{{}\begin{matrix}4a+b=1\\a=-2;b\ne3\end{matrix}\right.\Rightarrow\left\{{}\begin{matrix}4.\left(-2\right)+b=1\\a=-2;b\ne3\end{matrix}\right.\Rightarrow\left\{{}\begin{matrix}b=9\left(t/m\right)\\a=-2\end{matrix}\right.\)

Vậy \(d_3:y=-2x+9\)

#Rinz

Bình luận (0)
Thanhthanhhh
Xem chi tiết
Nguyễn Lê Phước Thịnh
17 tháng 5 2023 lúc 10:37

loading...

Bình luận (0)
♊Ngọc Hân♊
Xem chi tiết
Nam Duy
Xem chi tiết
Nguyễn Lê Phước Thịnh
20 tháng 12 2023 lúc 19:27

a: loading...

b: Phương trình hoành độ giao điểm là:

-2x+1=x-5

=>-2x-x=-5-1

=>-3x=-6

=>x=2

Thay x=2 vào y=x-5, ta được:

\(y=2-5=-3\)

Vậy: (d1) cắt (d2) tại A(2;-3)

c: (d1): y=x-5

=>x-y-5=0

Khoảng cách từ O(0;0) đến (d1) là:

\(d\left(O;\left(d1\right)\right)=\dfrac{\left|0\cdot1+0\cdot\left(-1\right)-5\right|}{\sqrt{1^2+\left(-1\right)^2}}=\dfrac{5}{\sqrt{2}}\)

(d2): y=-2x+1

=>y+2x-1=0

=>2x+y-1=0

Khoảng cách từ O đến (d2) là:

\(d\left(O;\left(d2\right)\right)=\dfrac{\left|0\cdot2+0\cdot1-1\right|}{\sqrt{2^2+1^2}}=\dfrac{1}{\sqrt{5}}\)

Bình luận (0)
James Pham
Xem chi tiết
Vũ Thúy Hằng
Xem chi tiết
Nguyễn Lê Phước Thịnh
18 tháng 12 2021 lúc 0:00

d: Để hai đường thẳng song song thì m=1

Bình luận (1)
Dương Tuấn Linh
Xem chi tiết
Nguyễn Lê Phước Thịnh
31 tháng 12 2021 lúc 10:20

b: Tọa độ giao điểm là:

\(\left\{{}\begin{matrix}3x=-2x+5\\y=3x\end{matrix}\right.\Leftrightarrow\left\{{}\begin{matrix}x=1\\y=3\end{matrix}\right.\)

Bình luận (0)
Logan
Xem chi tiết
Nguyễn Lê Phước Thịnh
3 tháng 9 2021 lúc 15:35

c: Vì (d2)//(d) nên \(a=-\dfrac{1}{2}\)

Thay x=-3 và y=0 vào \(y=\dfrac{-1}{2}x+b\), ta được:

\(b+\dfrac{3}{2}=0\)

hay \(b=-\dfrac{3}{2}\)

Bình luận (0)
Tú72 Cẩm
Xem chi tiết
Nguyễn Lê Phước Thịnh
25 tháng 11 2023 lúc 11:09

a: loading...

b: Phương trình hoành độ giao điểm là:

4x-2=-x+3

=>4x+x=3+2

=>5x=5

=>x=1

Thay x=1 vào y=-x+3, ta được:

\(y=-1+3=2\)

Vậy: M(1;2)

c: Gọi \(\alpha;\beta\) lần lượt là góc tạo bởi (d1),(d2) với trục Ox

(d1): y=4x-2

=>\(tan\alpha=4\)

=>\(\alpha=76^0\)

(d2): y=-x+3

=>\(tan\beta=-1\)

=>\(\beta=135^0\)

d: Thay y=6 vào (d1), ta được:

4x-2=6

=>4x=8

=>x=2

=>A(2;6)

Thay x=6/2=3 vào (d2), ta được:

\(y=-3+3=0\)

vậy: B(3;0)

Vì (d):y=ax+b đi qua A(2;6) và B(3;0) nên ta có hệ phương trình:

\(\left\{{}\begin{matrix}2a+b=6\\3a+b=0\end{matrix}\right.\)

=>\(\left\{{}\begin{matrix}2a+b-3a-b=6-0\\3a+b=0\end{matrix}\right.\Leftrightarrow\left\{{}\begin{matrix}-a=6\\b=-3a\end{matrix}\right.\)

=>\(\left\{{}\begin{matrix}a=-6\\b=-3\cdot\left(-6\right)=18\end{matrix}\right.\)

Vậy: (d): y=-6x+18

e: A(2;6); B(3;0); M(1;2)

\(AM=\sqrt{\left(1-2\right)^2+\left(2-6\right)^2}=\sqrt{17}\)

\(BM=\sqrt{\left(1-3\right)^2+\left(2-0\right)^2}=2\sqrt{2}\)

\(AB=\sqrt{\left(3-2\right)^2+\left(0-6\right)^2}=\sqrt{37}\)

Chu vi tam giác AMB là:

\(C_{AMB}=\sqrt{17}+2\sqrt{2}+\sqrt{37}\)

Xét ΔAMB có 

\(cosAMB=\dfrac{MA^2+MB^2-AB^2}{2\cdot MA\cdot MB}=\dfrac{17+8-37}{2\cdot2\sqrt{2}\cdot\sqrt{17}}=\dfrac{-3}{\sqrt{34}}\)

=>\(\widehat{AMB}\simeq121^0\) và \(sinAMB=\sqrt{1-\left(-\dfrac{3}{\sqrt{34}}\right)^2}=\dfrac{5}{\sqrt{34}}\)

Xét ΔAMB có

\(\dfrac{AB}{sinAMB}=\dfrac{AM}{sinABM}=\dfrac{BM}{sinBAM}\)

=>\(\dfrac{\sqrt{17}}{sinABM}=\dfrac{2\sqrt{2}}{sinBAM}=\sqrt{37}:\dfrac{5}{\sqrt{34}}\)

=>\(sinABM\simeq0,58;\widehat{BAM}\simeq0,4\)

=>\(\widehat{ABM}\simeq35^0;\widehat{BAM}\simeq24^0\)

Bình luận (0)